Single-use nuke-propelled mass driver spaceship












3












$begingroup$


So this is in the context where space battles are fought by ships that are little more than giant railguns (or other electromagnetic mass drivers), and therefore are fought at extreme ranges with very fast projectiles.



I have this rather fanciful idea of a single-use spaceship that is essentially a giant tube housing a projectile and a nuke, plus some aiming and targeting systems.
enter image description here
While the nuke would cause irreparable damage to the rest of the ship, the idea is that there would be enough time for the detonation to accelerate the projectile (or what remained of it) out of the barrel. The back end of the projectile would presumably have to be made from some ablative material.



Ignoring things like expense, practicality, targeting etc, I would like to know two things:




  1. Is this even a feasible design?


  2. How would one estimate the velocity of such a projectile, given the projectile mass $M_p$, the ship mass $M_s$ and the yield of the nuke used (which let's say for simplicity is of the range of current nuclear weapons, from 10 kt 100 t to 25 Mt)?


I am aware of Operation Plumbbob, where they detonated a 300 t nuke underground with a steel lid on the shaft weighing 900 kg. The lid was estimated at moving above 66,000 m/s, which is six times earth escape velocity.



EDIT: comments have pointed out that some sort of transference of thermal energy to gaseous kinetic energy is needed. Below is my revised picture.
enter image description here
Where A stands for ablator (not quite the right word I know, but I can't think of the right one off the top of my head), a mass of material that readily converts to a gas with high kinetic energy (I'm thinking of a slug of ice or solid ammonia, due to the hydrogen content) which then pushes on the projectile. I acknowledge that a megaton nuke is likely to disintegrate the entirety of the ship before any meaningful acceleration can be imparted on the projectile, but suspect that smaller ordnance will be better suited for creating appreciable acceleration on the projectile.










share|improve this question











$endgroup$












  • $begingroup$
    What's your definition of "extreme ranges?" Most people's concept of "extreme ranges" dramatically underestimates the actual distances between objects in space.
    $endgroup$
    – Cort Ammon
    6 hours ago










  • $begingroup$
    Like, tens, maybe even hundreds of feet. But seriously, I remember a quote saying that battles are fought at the furthest range possible. On that basis, the upper limit would be one of imaging the opposing spacecraft and ascertaining their position, velocity and orientation to obtain a firing solution. A softer limit would be the distance at which a ship can see an incoming projectile and reliably get out of the way. Both of those, I imagine, will be pretty extreme.
    $endgroup$
    – Ingolifs
    5 hours ago






  • 1




    $begingroup$
    Those would qualify as "extreme near ranges." If you're that close, it's clear you're trying to dock with a friendly ship. Real life space distances are much further. For perspective, the ISS maintains a 4km thick 25km wide disc of space -- if any one object is predicted to enter that region, we move the station. Of course, the objects are also traveling many kilometers every second!
    $endgroup$
    – Cort Ammon
    5 hours ago












  • $begingroup$
    the first sentence was ironic...
    $endgroup$
    – Ingolifs
    5 hours ago










  • $begingroup$
    Did you intend to be focusing on the extreme near ranges? At that distance, I'd recommend gently lobbing the nuke over to them and getting the heck outta there!
    $endgroup$
    – Cort Ammon
    5 hours ago


















3












$begingroup$


So this is in the context where space battles are fought by ships that are little more than giant railguns (or other electromagnetic mass drivers), and therefore are fought at extreme ranges with very fast projectiles.



I have this rather fanciful idea of a single-use spaceship that is essentially a giant tube housing a projectile and a nuke, plus some aiming and targeting systems.
enter image description here
While the nuke would cause irreparable damage to the rest of the ship, the idea is that there would be enough time for the detonation to accelerate the projectile (or what remained of it) out of the barrel. The back end of the projectile would presumably have to be made from some ablative material.



Ignoring things like expense, practicality, targeting etc, I would like to know two things:




  1. Is this even a feasible design?


  2. How would one estimate the velocity of such a projectile, given the projectile mass $M_p$, the ship mass $M_s$ and the yield of the nuke used (which let's say for simplicity is of the range of current nuclear weapons, from 10 kt 100 t to 25 Mt)?


I am aware of Operation Plumbbob, where they detonated a 300 t nuke underground with a steel lid on the shaft weighing 900 kg. The lid was estimated at moving above 66,000 m/s, which is six times earth escape velocity.



EDIT: comments have pointed out that some sort of transference of thermal energy to gaseous kinetic energy is needed. Below is my revised picture.
enter image description here
Where A stands for ablator (not quite the right word I know, but I can't think of the right one off the top of my head), a mass of material that readily converts to a gas with high kinetic energy (I'm thinking of a slug of ice or solid ammonia, due to the hydrogen content) which then pushes on the projectile. I acknowledge that a megaton nuke is likely to disintegrate the entirety of the ship before any meaningful acceleration can be imparted on the projectile, but suspect that smaller ordnance will be better suited for creating appreciable acceleration on the projectile.










share|improve this question











$endgroup$












  • $begingroup$
    What's your definition of "extreme ranges?" Most people's concept of "extreme ranges" dramatically underestimates the actual distances between objects in space.
    $endgroup$
    – Cort Ammon
    6 hours ago










  • $begingroup$
    Like, tens, maybe even hundreds of feet. But seriously, I remember a quote saying that battles are fought at the furthest range possible. On that basis, the upper limit would be one of imaging the opposing spacecraft and ascertaining their position, velocity and orientation to obtain a firing solution. A softer limit would be the distance at which a ship can see an incoming projectile and reliably get out of the way. Both of those, I imagine, will be pretty extreme.
    $endgroup$
    – Ingolifs
    5 hours ago






  • 1




    $begingroup$
    Those would qualify as "extreme near ranges." If you're that close, it's clear you're trying to dock with a friendly ship. Real life space distances are much further. For perspective, the ISS maintains a 4km thick 25km wide disc of space -- if any one object is predicted to enter that region, we move the station. Of course, the objects are also traveling many kilometers every second!
    $endgroup$
    – Cort Ammon
    5 hours ago












  • $begingroup$
    the first sentence was ironic...
    $endgroup$
    – Ingolifs
    5 hours ago










  • $begingroup$
    Did you intend to be focusing on the extreme near ranges? At that distance, I'd recommend gently lobbing the nuke over to them and getting the heck outta there!
    $endgroup$
    – Cort Ammon
    5 hours ago
















3












3








3





$begingroup$


So this is in the context where space battles are fought by ships that are little more than giant railguns (or other electromagnetic mass drivers), and therefore are fought at extreme ranges with very fast projectiles.



I have this rather fanciful idea of a single-use spaceship that is essentially a giant tube housing a projectile and a nuke, plus some aiming and targeting systems.
enter image description here
While the nuke would cause irreparable damage to the rest of the ship, the idea is that there would be enough time for the detonation to accelerate the projectile (or what remained of it) out of the barrel. The back end of the projectile would presumably have to be made from some ablative material.



Ignoring things like expense, practicality, targeting etc, I would like to know two things:




  1. Is this even a feasible design?


  2. How would one estimate the velocity of such a projectile, given the projectile mass $M_p$, the ship mass $M_s$ and the yield of the nuke used (which let's say for simplicity is of the range of current nuclear weapons, from 10 kt 100 t to 25 Mt)?


I am aware of Operation Plumbbob, where they detonated a 300 t nuke underground with a steel lid on the shaft weighing 900 kg. The lid was estimated at moving above 66,000 m/s, which is six times earth escape velocity.



EDIT: comments have pointed out that some sort of transference of thermal energy to gaseous kinetic energy is needed. Below is my revised picture.
enter image description here
Where A stands for ablator (not quite the right word I know, but I can't think of the right one off the top of my head), a mass of material that readily converts to a gas with high kinetic energy (I'm thinking of a slug of ice or solid ammonia, due to the hydrogen content) which then pushes on the projectile. I acknowledge that a megaton nuke is likely to disintegrate the entirety of the ship before any meaningful acceleration can be imparted on the projectile, but suspect that smaller ordnance will be better suited for creating appreciable acceleration on the projectile.










share|improve this question











$endgroup$




So this is in the context where space battles are fought by ships that are little more than giant railguns (or other electromagnetic mass drivers), and therefore are fought at extreme ranges with very fast projectiles.



I have this rather fanciful idea of a single-use spaceship that is essentially a giant tube housing a projectile and a nuke, plus some aiming and targeting systems.
enter image description here
While the nuke would cause irreparable damage to the rest of the ship, the idea is that there would be enough time for the detonation to accelerate the projectile (or what remained of it) out of the barrel. The back end of the projectile would presumably have to be made from some ablative material.



Ignoring things like expense, practicality, targeting etc, I would like to know two things:




  1. Is this even a feasible design?


  2. How would one estimate the velocity of such a projectile, given the projectile mass $M_p$, the ship mass $M_s$ and the yield of the nuke used (which let's say for simplicity is of the range of current nuclear weapons, from 10 kt 100 t to 25 Mt)?


I am aware of Operation Plumbbob, where they detonated a 300 t nuke underground with a steel lid on the shaft weighing 900 kg. The lid was estimated at moving above 66,000 m/s, which is six times earth escape velocity.



EDIT: comments have pointed out that some sort of transference of thermal energy to gaseous kinetic energy is needed. Below is my revised picture.
enter image description here
Where A stands for ablator (not quite the right word I know, but I can't think of the right one off the top of my head), a mass of material that readily converts to a gas with high kinetic energy (I'm thinking of a slug of ice or solid ammonia, due to the hydrogen content) which then pushes on the projectile. I acknowledge that a megaton nuke is likely to disintegrate the entirety of the ship before any meaningful acceleration can be imparted on the projectile, but suspect that smaller ordnance will be better suited for creating appreciable acceleration on the projectile.







nuclear-weapons space-combat kinetic-weapons






share|improve this question















share|improve this question













share|improve this question




share|improve this question








edited 3 hours ago







Ingolifs

















asked 7 hours ago









IngolifsIngolifs

1365




1365












  • $begingroup$
    What's your definition of "extreme ranges?" Most people's concept of "extreme ranges" dramatically underestimates the actual distances between objects in space.
    $endgroup$
    – Cort Ammon
    6 hours ago










  • $begingroup$
    Like, tens, maybe even hundreds of feet. But seriously, I remember a quote saying that battles are fought at the furthest range possible. On that basis, the upper limit would be one of imaging the opposing spacecraft and ascertaining their position, velocity and orientation to obtain a firing solution. A softer limit would be the distance at which a ship can see an incoming projectile and reliably get out of the way. Both of those, I imagine, will be pretty extreme.
    $endgroup$
    – Ingolifs
    5 hours ago






  • 1




    $begingroup$
    Those would qualify as "extreme near ranges." If you're that close, it's clear you're trying to dock with a friendly ship. Real life space distances are much further. For perspective, the ISS maintains a 4km thick 25km wide disc of space -- if any one object is predicted to enter that region, we move the station. Of course, the objects are also traveling many kilometers every second!
    $endgroup$
    – Cort Ammon
    5 hours ago












  • $begingroup$
    the first sentence was ironic...
    $endgroup$
    – Ingolifs
    5 hours ago










  • $begingroup$
    Did you intend to be focusing on the extreme near ranges? At that distance, I'd recommend gently lobbing the nuke over to them and getting the heck outta there!
    $endgroup$
    – Cort Ammon
    5 hours ago




















  • $begingroup$
    What's your definition of "extreme ranges?" Most people's concept of "extreme ranges" dramatically underestimates the actual distances between objects in space.
    $endgroup$
    – Cort Ammon
    6 hours ago










  • $begingroup$
    Like, tens, maybe even hundreds of feet. But seriously, I remember a quote saying that battles are fought at the furthest range possible. On that basis, the upper limit would be one of imaging the opposing spacecraft and ascertaining their position, velocity and orientation to obtain a firing solution. A softer limit would be the distance at which a ship can see an incoming projectile and reliably get out of the way. Both of those, I imagine, will be pretty extreme.
    $endgroup$
    – Ingolifs
    5 hours ago






  • 1




    $begingroup$
    Those would qualify as "extreme near ranges." If you're that close, it's clear you're trying to dock with a friendly ship. Real life space distances are much further. For perspective, the ISS maintains a 4km thick 25km wide disc of space -- if any one object is predicted to enter that region, we move the station. Of course, the objects are also traveling many kilometers every second!
    $endgroup$
    – Cort Ammon
    5 hours ago












  • $begingroup$
    the first sentence was ironic...
    $endgroup$
    – Ingolifs
    5 hours ago










  • $begingroup$
    Did you intend to be focusing on the extreme near ranges? At that distance, I'd recommend gently lobbing the nuke over to them and getting the heck outta there!
    $endgroup$
    – Cort Ammon
    5 hours ago


















$begingroup$
What's your definition of "extreme ranges?" Most people's concept of "extreme ranges" dramatically underestimates the actual distances between objects in space.
$endgroup$
– Cort Ammon
6 hours ago




$begingroup$
What's your definition of "extreme ranges?" Most people's concept of "extreme ranges" dramatically underestimates the actual distances between objects in space.
$endgroup$
– Cort Ammon
6 hours ago












$begingroup$
Like, tens, maybe even hundreds of feet. But seriously, I remember a quote saying that battles are fought at the furthest range possible. On that basis, the upper limit would be one of imaging the opposing spacecraft and ascertaining their position, velocity and orientation to obtain a firing solution. A softer limit would be the distance at which a ship can see an incoming projectile and reliably get out of the way. Both of those, I imagine, will be pretty extreme.
$endgroup$
– Ingolifs
5 hours ago




$begingroup$
Like, tens, maybe even hundreds of feet. But seriously, I remember a quote saying that battles are fought at the furthest range possible. On that basis, the upper limit would be one of imaging the opposing spacecraft and ascertaining their position, velocity and orientation to obtain a firing solution. A softer limit would be the distance at which a ship can see an incoming projectile and reliably get out of the way. Both of those, I imagine, will be pretty extreme.
$endgroup$
– Ingolifs
5 hours ago




1




1




$begingroup$
Those would qualify as "extreme near ranges." If you're that close, it's clear you're trying to dock with a friendly ship. Real life space distances are much further. For perspective, the ISS maintains a 4km thick 25km wide disc of space -- if any one object is predicted to enter that region, we move the station. Of course, the objects are also traveling many kilometers every second!
$endgroup$
– Cort Ammon
5 hours ago






$begingroup$
Those would qualify as "extreme near ranges." If you're that close, it's clear you're trying to dock with a friendly ship. Real life space distances are much further. For perspective, the ISS maintains a 4km thick 25km wide disc of space -- if any one object is predicted to enter that region, we move the station. Of course, the objects are also traveling many kilometers every second!
$endgroup$
– Cort Ammon
5 hours ago














$begingroup$
the first sentence was ironic...
$endgroup$
– Ingolifs
5 hours ago




$begingroup$
the first sentence was ironic...
$endgroup$
– Ingolifs
5 hours ago












$begingroup$
Did you intend to be focusing on the extreme near ranges? At that distance, I'd recommend gently lobbing the nuke over to them and getting the heck outta there!
$endgroup$
– Cort Ammon
5 hours ago






$begingroup$
Did you intend to be focusing on the extreme near ranges? At that distance, I'd recommend gently lobbing the nuke over to them and getting the heck outta there!
$endgroup$
– Cort Ammon
5 hours ago












4 Answers
4






active

oldest

votes


















5












$begingroup$

You seems to want speed calculations instead of the usual common sense explanation that one shouldn't use nuclear explosions to propel a projectile.... so, let's start, I guess.



First, we have conservation of momentum: $mathbf p_s + mathbf p_p = 0$, assuming we are in the reference frame that the ship was initially at rest. Second, we have conservation of energy:
$$
U = K_S + K_P + Q
$$



Where $U$ is the total energy of the bomb, $K_P$ is the kinetic energy of the projectile, and $K_S$ the kinetic energy of the ship (or the total sum of kinetic energy of all the wreckage left by the ship). And, lastly, $Q$ is the thermal energy.



Now, it is important to notice that these two equations are valid for any and all given times. Because of that, we'll take a time where the projectile is launched, but the nuclear exposion didn't have enough time to completely destroy the ship itself (by too much). Thus, the ship is still in one piece, and thus, we are talking about a two-body system. That will greatly simplify everything.



Let $v_p, v_s$ the velocity of projectile and ship. By momentum conservation, we know:
$$
m_s v_s = -m_p v_p
quadimpliesquad
v_s = -frac{m_p}{m_s}v_p
$$



Thus, the energy:
$$
K =
frac{1}{2}m_s v_s^2 + frac{1}{2}m_p v_p^2 =
frac{1}{2}m_sleft(-frac{m_p}{m_s}v_pright)^2 + frac{1}{2}m_p^2
$$



Thus:
$$
K = frac{1}{2}left(m_s + frac{m_p^2}{m_s}right)v_p^2
= frac{1}{2}m_sleft[1 + left(frac{m_p}{m_s}v_pright)^2right]
$$



There, the projectile velocity is simply:
$$
v_p = frac{m_s}{m_p}sqrt{frac{2K}{m_s} - 1}
$$



Where $U = Q + K$. Now it would be a matter of estimating the ratio between kinetic and total energy available, or, the efficiency $eta = K/U$ of your transformation between bomb's energy to kinetic energy.



Plugging numbers: If $U = 25MT = 1.046cdot 10^{17}J$, setting $eta = 0.1%$, this means $K = 1.046cdot 10^{14}J$. If $m_s = 10^9 Kg$ and $m_p = 10^3 Kg$, then: $v_p = 0.48245c$. Almost close to speed of light. Be aware that all of this ignores the obvious problems of having a nuclear explosion to propel a bullet: if you have those problems solved, then this calculations gives you a rough speed estimate of the projectile. And keep in mind I completely guessed $eta$ (such value will depend on the actual mechanism you have to transform the bomb's energy to kinetic energy). Besides, $eta$ could be muuuch MUUUCH less than the guessed value (it probably is).






share|improve this answer









$endgroup$













  • $begingroup$
    Also, these calculations are not relativistic. I might be perhaps persuaded to make a relativistic version of them if everyone wants it...
    $endgroup$
    – Physicist137
    2 hours ago












  • $begingroup$
    That's probably not necessary, Between you and JBH, a very high upper bound for the velocity of the projectile has been established. What I'm starting to suspect is even if you can fling a projectile (that isn't now a glob of molten metal or plasma spray) using a nuke, any asymmetry in the detonation will probably result in the gun being kicked off-target, resulting in the projectile missing completely.
    $endgroup$
    – Ingolifs
    1 hour ago










  • $begingroup$
    @Ingolifs I was doing the relativistic calculations in a paper (just because I am too curious). I planned to only post the result. But these are huuuge. After almost half page were completely filled, there were a lot more calculations to do, so, I gave up. :). Also, notice that, a ordinary weapon is much like your design, except, they use a chemical explosion instead of a nuclear one -- and bullet weapons can be designed to be very accurate. Thus, maybe there's a way to design the nuclear one, where it could be accurate.
    $endgroup$
    – Physicist137
    1 hour ago










  • $begingroup$
    I love it. Thank you!
    $endgroup$
    – JBH
    1 hour ago



















3












$begingroup$

Tim's right, but let's have some fun explaining why.



Assumptions




  • When you said "railgun" what you meant was "slug thrower." A railgun is something specific, and you're not describing it.


  • The "gun" can't be damaged by the nuke. This is an experiment, after all.


  • Neither can the shell.


  • The practicality of the one-use-incredibly-expensive-hard-to-aim gun is ignored.


  • The "gun" has a mass at least 1,000X of the shell. Thus, 99.9% of the final velocity belongs to the shell, while only 0.1% of the final velocity belongs to the gun (which enters a decaying orbit, devastating the southern continent of your world, but that's a story for another day). Anyway, what this does is let me reasonably assume all the energy is used to push the shell.


  • Here's the whopper: we're going to assume that when they measure the explosive capability of a nuke in terms of tons of TNT, they really mean it (and the angels weep over the magnitude of this lie... but we'll get to that).



The equation of choice is F=mA. Let's be outrageous.




  • The charge is equivalent to the Tsar Bomba: 50 megatons or 210 peta-joules. So, F = 210x1015.

  • The shell is a useful 10,000 Kg. So, m = 1x104.

  • A (acceleration) = 210x1011 m/s2.


It has no resistance (space...) and there's no more force acting upon it. So our final velocity is darn close to twice the speed of light.



Which obviously can't happen



And that's why Tim's answer is correct: puff of light and heat.



OK, what's the problem?



A whomping large amount of a nuke's energy has nothing at all to do with kinetic energy. It's radiation. Oversimplifying something awful, you only have the mass of the charge to work with in terms of expanding gas (kinetic force) and a lot (if not all) of it just got converted to pure energy.



Most of the concussive force of the nuclear explosions we're familiar with are (a) the conversion of surrounding mass (air, soil, houses, people) into expanding plasma and (b) the shock created when the vacuum caused by the explosion collapses.



Which is a fancy way of saying you don't actually have that much to work with. Since you only have whatever convert-to-plasma materials are in the nuke itself (gun and shell can't be damaged by the nuke!), your gun will glow white hot1 while the equally white hot shell kinda blurps out of the muzzle and lazily meanders away.



If I got this wrong, please correct me!





Edit:



Physicist137 pointed out the flaw in my math, and I'm grateful! I had fun, but go upvote his math! I'm still happy with my conclusions, but his ~0.5c answer is a whole lot more realistic than my 2c analysis!





1And it'll probably stay white hot for decades. Space is a terrible place to dump heat.






share|improve this answer











$endgroup$









  • 1




    $begingroup$
    You got me with Tsar Bomba.
    $endgroup$
    – Mr.J
    4 hours ago










  • $begingroup$
    "So our final velocity is darn close to twice the speed of light. Which obviously can't happen" Ok so this part is fallacious for a couple of reasons, 1) because it ignores relativity, and 2) because it implies a derived speed greater than c (while ignoring relativity) counts as reducto ad absurdium. The ship is not indestructible, neither is the projectile. My question isn't about how badly the craft will blow up (it's single use, so it's already written off), but whether you can get significant velocity on the projectile that comes out of it.
    $endgroup$
    – Ingolifs
    3 hours ago










  • $begingroup$
    @Ingolifs You're not having nearly enough fun. You're missing mass between the slug and the nuke that can be converted to plasma. However, what you haven't figured out is that almost anything is more useful in space than a nuke-powered slug thrower. Especially a one-use thrower.
    $endgroup$
    – JBH
    3 hours ago






  • 2




    $begingroup$
    Hmm. Maybe I need to update the picture I provided with a mass of something ablative between nuke and projectile.
    $endgroup$
    – Ingolifs
    3 hours ago










  • $begingroup$
    I am afraid this is not correct. You are equaling a force (from newton's second law) with an energy (the bomb), and you cannot do that.
    $endgroup$
    – Physicist137
    2 hours ago





















1












$begingroup$

There's a simple problem with this: shock and vibe.



Your projectile has to travel a very long distance before hitting. Your specification of "around the limit of detection by IR" is sufficient to be quite certain you will never hit your enemy with a dumb slug. You simply can't aim the gun that accurately. This is doubly true for the fact that you're about to set off a nuclear explosion (probably well more than doubly true).



This means you're certainly using a smart weapon, with onboard propellants, sensors, and computers, much like we see in all of our own ballistic defense programs on planet Earth.



So here's the real issue. Launching things fast (space fast) is hard. You have to accelerate rather quickly. Shock and vibe issues during launch are astonishingly difficult to deal with. The harder you accelerate, the nastier the requirements. Navy rail guns face serious challenges in hardening smart rounds so that it survives being fired (tests are typically done with dumb Tungsten slugs today, but it is recognized that they will need to be guided eventually to strike distant targets). You have invented a new level of "difficult launch conditions." Railguns can maintain a maximized shock and vibe for the length of the rail far better than a simple nuclear gun can.



Launch Conditions: Must survive being at ground zero of a 25MT thermonuclear bomb detonation, in a confined space designed to maximize the impact of the thermonuclear bomb on the projectile.



Ouch!



What you really want to do is stick to railguns. If you design a projectile that can survive the forces of a nuclear bomb, you immediately put that same projectile into a railgun, and get more performance out of it (railguns can accelerate linearly along the entire length of the rails, while this gun peters out as the pressure wave weakens)






share|improve this answer









$endgroup$













  • $begingroup$
    Good answer. There is a 'combat regime' I guess you'd call it, where you can see your enemy but do not have the ability to hit them, because the time-of-flight of your projectiles is longer than the time needed to get out of the way. If you make your projectile faster, you have a better chance of hitting, providing you do not sacrifice accuracy. Your point about projectiles needing guidance in any case is a good one. That will always limit the acceleration of the projectile to what the guidance computer system can survive. In which case, using the nukes for Orion drive would be better.
    $endgroup$
    – Ingolifs
    1 hour ago










  • $begingroup$
    If I remember my rules for intercepts correctly, faster doesn't actually make it much better. Reaction times are important, and faster will make it better with respect to reactions, but once you actually get to the demanding task of moving your ship out of the way of the round, faster no lnoger helps. They have less time to move, but you have less time to respond to their movement, so it cancels out.
    $endgroup$
    – Cort Ammon
    1 hour ago



















-1












$begingroup$

Your single use ship will disappear in a puff of light and heat.



The problem with what you're describing is that a nuclear bomb is exactly the wrong type of energy source for a rail gun as it generates thermal heat and radiation, not kinetic energy, in space.



We've all read about the blast waves off nuclear explosions and how powerful they are; there is even some research into this that shows that blast (kinetic) energy comprises roughly half the energy released in a nuclear explosion. The trouble is, that blast energy is essentially the atmosphere super heating from the thermal energy released, and therefore rapidly increasing in pressure in an unconfined space.



Nuclear bombs are not designed to release kinetic energy; they're designed to release thermal energy. This can still work because of a technique known as Ablative propulsion, which amounts to superheating a surface of an object to a point where the surface becomes a plasma that pushes the object rapidly in the opposite direction to that in which the heated surface is facing.



This kind of thinking about propulsion seems to be behind Nuclear Pulse Propulsion theoretical engine systems like that originally planned for Project Orion. My reading on this to date seems to indicate that the plan was to detonate nuclear bombs outside the ship and have the blast hit a pusher plate on the back of the ship, pushing it in that direction. What is unclear in all those mainstream articles is how that blast wave is generated but the best I can come up with is that the mass of the bomb itself would become a superheated plasma that would push against the rear plate of the ship. The closest I could come up with in terms of reference to how nuclear energy can be converted to kinetic energy in space was a reference to Electrical v Direct Thrust on fusion drives, but even it talks about the plasma having to be contained to sustain the reaction which to my mind seems contradictory.



If this isn't the thinking behind nuclear pulse theory and you know what is, leave it in comments and I'll integrate it into my answer with full credits. But back to the matter at hand...



In space, there's no atmosphere to heat up, so no localised pressure differential, therefore no blast wave. What WILL happen is that the heat energy will essentially vapourise your ship and the projectile through heating them all equally.



To be fair, this will result in the aforementioned ablative propulsion for your rail projectile; as the weak point in the containment, it's going to break off (or out) first but without running some serious math I have no idea just how quickly it will escape the barrel, how much of it will be left to hit the target, or even if it would escape the barrel at all before the thermal energy expands it (and contracts the barrel) before it escapes.



Before anyone says it, yes, the barrel will constrict. As the metal expands, the inner metal of the barrel also expands, meaning that the hole gets smaller through swelling.



Guns like cannon, pistols, rifles and rail guns work through the kinetic energy of expanding gases. The idea is that gunpowder, when burnt, releases a massive amount of hot gas into a confined area, forcing out the weakest point of the containment chamber, which just happens to be the bullet. This projects it suddenly in the direction of your choosing. Yes, burnt gunpowder is hot, but it's not the heat that shoots out the bullet; it's the expanding gases.



In space, your setup has no atmosphere around it to expand, and even if it did, the kinetic energy transferred to it only represents about half the energy that a nuclear device can produce, which makes it an inefficient energy source for what you're trying to do, and would require such a sturdy, heavy ship to withstand the vapourisation long enough to transfer the bulk of the energy to the projectile so as to make it uneconomic as an investment.



For my money, you'd be far better off investing in a nuclear powerplant that feeds electrical capacitors that are capable of releasing a sudden energy burst of electricity to propel your bullet. Far more efficient process of converting thermal to kinetic energy (via temporarily being electricity), reusable and less dangerous to the rest of your fleet by not creating debris fields with every shot.






share|improve this answer











$endgroup$













  • $begingroup$
    While I too question its practicality, is this not roughly analogous to the nuclear pulse rocket concept?
    $endgroup$
    – Cort Ammon
    6 hours ago










  • $begingroup$
    @CortAmmon you're right; I'd forgotten about that until you mentioned it. I just did a quick check on it and most of the material I can find on this talks about specific impulses, but doesn't actually go into detail about how the blast wave in space actually works. It seems to be related to the surrounding mass being converted to plasma. I'll do some more research and edit accordingly within the next couple of hours.
    $endgroup$
    – Tim B II
    6 hours ago










  • $begingroup$
    I'm not convinced by this answer. Operation Plumbbob has shown that such a thing is at least possible, though it may require a separation of gas between the nuclear explosive and the projectile, and probably a much weaker nuke than the likes of Fat Man (let alone Tsar Bomba).
    $endgroup$
    – Ingolifs
    3 hours ago










  • $begingroup$
    @Ingolifs the trouble with comparing Operation Plumbbob to this scenario is that it was conducted on Earth, not space. Earth has an atmosphere that can be heated as an expanding gas and plasma, space doesn't have a medium in which to propagate the blast wave.
    $endgroup$
    – Tim B II
    2 hours ago










  • $begingroup$
    I've edited my original question to include a mass of material between the projectile and the nuclear explosive, that vaporises into gas and plasma.
    $endgroup$
    – Ingolifs
    2 hours ago











Your Answer





StackExchange.ifUsing("editor", function () {
return StackExchange.using("mathjaxEditing", function () {
StackExchange.MarkdownEditor.creationCallbacks.add(function (editor, postfix) {
StackExchange.mathjaxEditing.prepareWmdForMathJax(editor, postfix, [["$", "$"], ["\\(","\\)"]]);
});
});
}, "mathjax-editing");

StackExchange.ready(function() {
var channelOptions = {
tags: "".split(" "),
id: "579"
};
initTagRenderer("".split(" "), "".split(" "), channelOptions);

StackExchange.using("externalEditor", function() {
// Have to fire editor after snippets, if snippets enabled
if (StackExchange.settings.snippets.snippetsEnabled) {
StackExchange.using("snippets", function() {
createEditor();
});
}
else {
createEditor();
}
});

function createEditor() {
StackExchange.prepareEditor({
heartbeatType: 'answer',
autoActivateHeartbeat: false,
convertImagesToLinks: false,
noModals: true,
showLowRepImageUploadWarning: true,
reputationToPostImages: null,
bindNavPrevention: true,
postfix: "",
imageUploader: {
brandingHtml: "Powered by u003ca class="icon-imgur-white" href="https://imgur.com/"u003eu003c/au003e",
contentPolicyHtml: "User contributions licensed under u003ca href="https://creativecommons.org/licenses/by-sa/3.0/"u003ecc by-sa 3.0 with attribution requiredu003c/au003e u003ca href="https://stackoverflow.com/legal/content-policy"u003e(content policy)u003c/au003e",
allowUrls: true
},
noCode: true, onDemand: true,
discardSelector: ".discard-answer"
,immediatelyShowMarkdownHelp:true
});


}
});














draft saved

draft discarded


















StackExchange.ready(
function () {
StackExchange.openid.initPostLogin('.new-post-login', 'https%3a%2f%2fworldbuilding.stackexchange.com%2fquestions%2f136933%2fsingle-use-nuke-propelled-mass-driver-spaceship%23new-answer', 'question_page');
}
);

Post as a guest















Required, but never shown

























4 Answers
4






active

oldest

votes








4 Answers
4






active

oldest

votes









active

oldest

votes






active

oldest

votes









5












$begingroup$

You seems to want speed calculations instead of the usual common sense explanation that one shouldn't use nuclear explosions to propel a projectile.... so, let's start, I guess.



First, we have conservation of momentum: $mathbf p_s + mathbf p_p = 0$, assuming we are in the reference frame that the ship was initially at rest. Second, we have conservation of energy:
$$
U = K_S + K_P + Q
$$



Where $U$ is the total energy of the bomb, $K_P$ is the kinetic energy of the projectile, and $K_S$ the kinetic energy of the ship (or the total sum of kinetic energy of all the wreckage left by the ship). And, lastly, $Q$ is the thermal energy.



Now, it is important to notice that these two equations are valid for any and all given times. Because of that, we'll take a time where the projectile is launched, but the nuclear exposion didn't have enough time to completely destroy the ship itself (by too much). Thus, the ship is still in one piece, and thus, we are talking about a two-body system. That will greatly simplify everything.



Let $v_p, v_s$ the velocity of projectile and ship. By momentum conservation, we know:
$$
m_s v_s = -m_p v_p
quadimpliesquad
v_s = -frac{m_p}{m_s}v_p
$$



Thus, the energy:
$$
K =
frac{1}{2}m_s v_s^2 + frac{1}{2}m_p v_p^2 =
frac{1}{2}m_sleft(-frac{m_p}{m_s}v_pright)^2 + frac{1}{2}m_p^2
$$



Thus:
$$
K = frac{1}{2}left(m_s + frac{m_p^2}{m_s}right)v_p^2
= frac{1}{2}m_sleft[1 + left(frac{m_p}{m_s}v_pright)^2right]
$$



There, the projectile velocity is simply:
$$
v_p = frac{m_s}{m_p}sqrt{frac{2K}{m_s} - 1}
$$



Where $U = Q + K$. Now it would be a matter of estimating the ratio between kinetic and total energy available, or, the efficiency $eta = K/U$ of your transformation between bomb's energy to kinetic energy.



Plugging numbers: If $U = 25MT = 1.046cdot 10^{17}J$, setting $eta = 0.1%$, this means $K = 1.046cdot 10^{14}J$. If $m_s = 10^9 Kg$ and $m_p = 10^3 Kg$, then: $v_p = 0.48245c$. Almost close to speed of light. Be aware that all of this ignores the obvious problems of having a nuclear explosion to propel a bullet: if you have those problems solved, then this calculations gives you a rough speed estimate of the projectile. And keep in mind I completely guessed $eta$ (such value will depend on the actual mechanism you have to transform the bomb's energy to kinetic energy). Besides, $eta$ could be muuuch MUUUCH less than the guessed value (it probably is).






share|improve this answer









$endgroup$













  • $begingroup$
    Also, these calculations are not relativistic. I might be perhaps persuaded to make a relativistic version of them if everyone wants it...
    $endgroup$
    – Physicist137
    2 hours ago












  • $begingroup$
    That's probably not necessary, Between you and JBH, a very high upper bound for the velocity of the projectile has been established. What I'm starting to suspect is even if you can fling a projectile (that isn't now a glob of molten metal or plasma spray) using a nuke, any asymmetry in the detonation will probably result in the gun being kicked off-target, resulting in the projectile missing completely.
    $endgroup$
    – Ingolifs
    1 hour ago










  • $begingroup$
    @Ingolifs I was doing the relativistic calculations in a paper (just because I am too curious). I planned to only post the result. But these are huuuge. After almost half page were completely filled, there were a lot more calculations to do, so, I gave up. :). Also, notice that, a ordinary weapon is much like your design, except, they use a chemical explosion instead of a nuclear one -- and bullet weapons can be designed to be very accurate. Thus, maybe there's a way to design the nuclear one, where it could be accurate.
    $endgroup$
    – Physicist137
    1 hour ago










  • $begingroup$
    I love it. Thank you!
    $endgroup$
    – JBH
    1 hour ago
















5












$begingroup$

You seems to want speed calculations instead of the usual common sense explanation that one shouldn't use nuclear explosions to propel a projectile.... so, let's start, I guess.



First, we have conservation of momentum: $mathbf p_s + mathbf p_p = 0$, assuming we are in the reference frame that the ship was initially at rest. Second, we have conservation of energy:
$$
U = K_S + K_P + Q
$$



Where $U$ is the total energy of the bomb, $K_P$ is the kinetic energy of the projectile, and $K_S$ the kinetic energy of the ship (or the total sum of kinetic energy of all the wreckage left by the ship). And, lastly, $Q$ is the thermal energy.



Now, it is important to notice that these two equations are valid for any and all given times. Because of that, we'll take a time where the projectile is launched, but the nuclear exposion didn't have enough time to completely destroy the ship itself (by too much). Thus, the ship is still in one piece, and thus, we are talking about a two-body system. That will greatly simplify everything.



Let $v_p, v_s$ the velocity of projectile and ship. By momentum conservation, we know:
$$
m_s v_s = -m_p v_p
quadimpliesquad
v_s = -frac{m_p}{m_s}v_p
$$



Thus, the energy:
$$
K =
frac{1}{2}m_s v_s^2 + frac{1}{2}m_p v_p^2 =
frac{1}{2}m_sleft(-frac{m_p}{m_s}v_pright)^2 + frac{1}{2}m_p^2
$$



Thus:
$$
K = frac{1}{2}left(m_s + frac{m_p^2}{m_s}right)v_p^2
= frac{1}{2}m_sleft[1 + left(frac{m_p}{m_s}v_pright)^2right]
$$



There, the projectile velocity is simply:
$$
v_p = frac{m_s}{m_p}sqrt{frac{2K}{m_s} - 1}
$$



Where $U = Q + K$. Now it would be a matter of estimating the ratio between kinetic and total energy available, or, the efficiency $eta = K/U$ of your transformation between bomb's energy to kinetic energy.



Plugging numbers: If $U = 25MT = 1.046cdot 10^{17}J$, setting $eta = 0.1%$, this means $K = 1.046cdot 10^{14}J$. If $m_s = 10^9 Kg$ and $m_p = 10^3 Kg$, then: $v_p = 0.48245c$. Almost close to speed of light. Be aware that all of this ignores the obvious problems of having a nuclear explosion to propel a bullet: if you have those problems solved, then this calculations gives you a rough speed estimate of the projectile. And keep in mind I completely guessed $eta$ (such value will depend on the actual mechanism you have to transform the bomb's energy to kinetic energy). Besides, $eta$ could be muuuch MUUUCH less than the guessed value (it probably is).






share|improve this answer









$endgroup$













  • $begingroup$
    Also, these calculations are not relativistic. I might be perhaps persuaded to make a relativistic version of them if everyone wants it...
    $endgroup$
    – Physicist137
    2 hours ago












  • $begingroup$
    That's probably not necessary, Between you and JBH, a very high upper bound for the velocity of the projectile has been established. What I'm starting to suspect is even if you can fling a projectile (that isn't now a glob of molten metal or plasma spray) using a nuke, any asymmetry in the detonation will probably result in the gun being kicked off-target, resulting in the projectile missing completely.
    $endgroup$
    – Ingolifs
    1 hour ago










  • $begingroup$
    @Ingolifs I was doing the relativistic calculations in a paper (just because I am too curious). I planned to only post the result. But these are huuuge. After almost half page were completely filled, there were a lot more calculations to do, so, I gave up. :). Also, notice that, a ordinary weapon is much like your design, except, they use a chemical explosion instead of a nuclear one -- and bullet weapons can be designed to be very accurate. Thus, maybe there's a way to design the nuclear one, where it could be accurate.
    $endgroup$
    – Physicist137
    1 hour ago










  • $begingroup$
    I love it. Thank you!
    $endgroup$
    – JBH
    1 hour ago














5












5








5





$begingroup$

You seems to want speed calculations instead of the usual common sense explanation that one shouldn't use nuclear explosions to propel a projectile.... so, let's start, I guess.



First, we have conservation of momentum: $mathbf p_s + mathbf p_p = 0$, assuming we are in the reference frame that the ship was initially at rest. Second, we have conservation of energy:
$$
U = K_S + K_P + Q
$$



Where $U$ is the total energy of the bomb, $K_P$ is the kinetic energy of the projectile, and $K_S$ the kinetic energy of the ship (or the total sum of kinetic energy of all the wreckage left by the ship). And, lastly, $Q$ is the thermal energy.



Now, it is important to notice that these two equations are valid for any and all given times. Because of that, we'll take a time where the projectile is launched, but the nuclear exposion didn't have enough time to completely destroy the ship itself (by too much). Thus, the ship is still in one piece, and thus, we are talking about a two-body system. That will greatly simplify everything.



Let $v_p, v_s$ the velocity of projectile and ship. By momentum conservation, we know:
$$
m_s v_s = -m_p v_p
quadimpliesquad
v_s = -frac{m_p}{m_s}v_p
$$



Thus, the energy:
$$
K =
frac{1}{2}m_s v_s^2 + frac{1}{2}m_p v_p^2 =
frac{1}{2}m_sleft(-frac{m_p}{m_s}v_pright)^2 + frac{1}{2}m_p^2
$$



Thus:
$$
K = frac{1}{2}left(m_s + frac{m_p^2}{m_s}right)v_p^2
= frac{1}{2}m_sleft[1 + left(frac{m_p}{m_s}v_pright)^2right]
$$



There, the projectile velocity is simply:
$$
v_p = frac{m_s}{m_p}sqrt{frac{2K}{m_s} - 1}
$$



Where $U = Q + K$. Now it would be a matter of estimating the ratio between kinetic and total energy available, or, the efficiency $eta = K/U$ of your transformation between bomb's energy to kinetic energy.



Plugging numbers: If $U = 25MT = 1.046cdot 10^{17}J$, setting $eta = 0.1%$, this means $K = 1.046cdot 10^{14}J$. If $m_s = 10^9 Kg$ and $m_p = 10^3 Kg$, then: $v_p = 0.48245c$. Almost close to speed of light. Be aware that all of this ignores the obvious problems of having a nuclear explosion to propel a bullet: if you have those problems solved, then this calculations gives you a rough speed estimate of the projectile. And keep in mind I completely guessed $eta$ (such value will depend on the actual mechanism you have to transform the bomb's energy to kinetic energy). Besides, $eta$ could be muuuch MUUUCH less than the guessed value (it probably is).






share|improve this answer









$endgroup$



You seems to want speed calculations instead of the usual common sense explanation that one shouldn't use nuclear explosions to propel a projectile.... so, let's start, I guess.



First, we have conservation of momentum: $mathbf p_s + mathbf p_p = 0$, assuming we are in the reference frame that the ship was initially at rest. Second, we have conservation of energy:
$$
U = K_S + K_P + Q
$$



Where $U$ is the total energy of the bomb, $K_P$ is the kinetic energy of the projectile, and $K_S$ the kinetic energy of the ship (or the total sum of kinetic energy of all the wreckage left by the ship). And, lastly, $Q$ is the thermal energy.



Now, it is important to notice that these two equations are valid for any and all given times. Because of that, we'll take a time where the projectile is launched, but the nuclear exposion didn't have enough time to completely destroy the ship itself (by too much). Thus, the ship is still in one piece, and thus, we are talking about a two-body system. That will greatly simplify everything.



Let $v_p, v_s$ the velocity of projectile and ship. By momentum conservation, we know:
$$
m_s v_s = -m_p v_p
quadimpliesquad
v_s = -frac{m_p}{m_s}v_p
$$



Thus, the energy:
$$
K =
frac{1}{2}m_s v_s^2 + frac{1}{2}m_p v_p^2 =
frac{1}{2}m_sleft(-frac{m_p}{m_s}v_pright)^2 + frac{1}{2}m_p^2
$$



Thus:
$$
K = frac{1}{2}left(m_s + frac{m_p^2}{m_s}right)v_p^2
= frac{1}{2}m_sleft[1 + left(frac{m_p}{m_s}v_pright)^2right]
$$



There, the projectile velocity is simply:
$$
v_p = frac{m_s}{m_p}sqrt{frac{2K}{m_s} - 1}
$$



Where $U = Q + K$. Now it would be a matter of estimating the ratio between kinetic and total energy available, or, the efficiency $eta = K/U$ of your transformation between bomb's energy to kinetic energy.



Plugging numbers: If $U = 25MT = 1.046cdot 10^{17}J$, setting $eta = 0.1%$, this means $K = 1.046cdot 10^{14}J$. If $m_s = 10^9 Kg$ and $m_p = 10^3 Kg$, then: $v_p = 0.48245c$. Almost close to speed of light. Be aware that all of this ignores the obvious problems of having a nuclear explosion to propel a bullet: if you have those problems solved, then this calculations gives you a rough speed estimate of the projectile. And keep in mind I completely guessed $eta$ (such value will depend on the actual mechanism you have to transform the bomb's energy to kinetic energy). Besides, $eta$ could be muuuch MUUUCH less than the guessed value (it probably is).







share|improve this answer












share|improve this answer



share|improve this answer










answered 2 hours ago









Physicist137Physicist137

69638




69638












  • $begingroup$
    Also, these calculations are not relativistic. I might be perhaps persuaded to make a relativistic version of them if everyone wants it...
    $endgroup$
    – Physicist137
    2 hours ago












  • $begingroup$
    That's probably not necessary, Between you and JBH, a very high upper bound for the velocity of the projectile has been established. What I'm starting to suspect is even if you can fling a projectile (that isn't now a glob of molten metal or plasma spray) using a nuke, any asymmetry in the detonation will probably result in the gun being kicked off-target, resulting in the projectile missing completely.
    $endgroup$
    – Ingolifs
    1 hour ago










  • $begingroup$
    @Ingolifs I was doing the relativistic calculations in a paper (just because I am too curious). I planned to only post the result. But these are huuuge. After almost half page were completely filled, there were a lot more calculations to do, so, I gave up. :). Also, notice that, a ordinary weapon is much like your design, except, they use a chemical explosion instead of a nuclear one -- and bullet weapons can be designed to be very accurate. Thus, maybe there's a way to design the nuclear one, where it could be accurate.
    $endgroup$
    – Physicist137
    1 hour ago










  • $begingroup$
    I love it. Thank you!
    $endgroup$
    – JBH
    1 hour ago


















  • $begingroup$
    Also, these calculations are not relativistic. I might be perhaps persuaded to make a relativistic version of them if everyone wants it...
    $endgroup$
    – Physicist137
    2 hours ago












  • $begingroup$
    That's probably not necessary, Between you and JBH, a very high upper bound for the velocity of the projectile has been established. What I'm starting to suspect is even if you can fling a projectile (that isn't now a glob of molten metal or plasma spray) using a nuke, any asymmetry in the detonation will probably result in the gun being kicked off-target, resulting in the projectile missing completely.
    $endgroup$
    – Ingolifs
    1 hour ago










  • $begingroup$
    @Ingolifs I was doing the relativistic calculations in a paper (just because I am too curious). I planned to only post the result. But these are huuuge. After almost half page were completely filled, there were a lot more calculations to do, so, I gave up. :). Also, notice that, a ordinary weapon is much like your design, except, they use a chemical explosion instead of a nuclear one -- and bullet weapons can be designed to be very accurate. Thus, maybe there's a way to design the nuclear one, where it could be accurate.
    $endgroup$
    – Physicist137
    1 hour ago










  • $begingroup$
    I love it. Thank you!
    $endgroup$
    – JBH
    1 hour ago
















$begingroup$
Also, these calculations are not relativistic. I might be perhaps persuaded to make a relativistic version of them if everyone wants it...
$endgroup$
– Physicist137
2 hours ago






$begingroup$
Also, these calculations are not relativistic. I might be perhaps persuaded to make a relativistic version of them if everyone wants it...
$endgroup$
– Physicist137
2 hours ago














$begingroup$
That's probably not necessary, Between you and JBH, a very high upper bound for the velocity of the projectile has been established. What I'm starting to suspect is even if you can fling a projectile (that isn't now a glob of molten metal or plasma spray) using a nuke, any asymmetry in the detonation will probably result in the gun being kicked off-target, resulting in the projectile missing completely.
$endgroup$
– Ingolifs
1 hour ago




$begingroup$
That's probably not necessary, Between you and JBH, a very high upper bound for the velocity of the projectile has been established. What I'm starting to suspect is even if you can fling a projectile (that isn't now a glob of molten metal or plasma spray) using a nuke, any asymmetry in the detonation will probably result in the gun being kicked off-target, resulting in the projectile missing completely.
$endgroup$
– Ingolifs
1 hour ago












$begingroup$
@Ingolifs I was doing the relativistic calculations in a paper (just because I am too curious). I planned to only post the result. But these are huuuge. After almost half page were completely filled, there were a lot more calculations to do, so, I gave up. :). Also, notice that, a ordinary weapon is much like your design, except, they use a chemical explosion instead of a nuclear one -- and bullet weapons can be designed to be very accurate. Thus, maybe there's a way to design the nuclear one, where it could be accurate.
$endgroup$
– Physicist137
1 hour ago




$begingroup$
@Ingolifs I was doing the relativistic calculations in a paper (just because I am too curious). I planned to only post the result. But these are huuuge. After almost half page were completely filled, there were a lot more calculations to do, so, I gave up. :). Also, notice that, a ordinary weapon is much like your design, except, they use a chemical explosion instead of a nuclear one -- and bullet weapons can be designed to be very accurate. Thus, maybe there's a way to design the nuclear one, where it could be accurate.
$endgroup$
– Physicist137
1 hour ago












$begingroup$
I love it. Thank you!
$endgroup$
– JBH
1 hour ago




$begingroup$
I love it. Thank you!
$endgroup$
– JBH
1 hour ago











3












$begingroup$

Tim's right, but let's have some fun explaining why.



Assumptions




  • When you said "railgun" what you meant was "slug thrower." A railgun is something specific, and you're not describing it.


  • The "gun" can't be damaged by the nuke. This is an experiment, after all.


  • Neither can the shell.


  • The practicality of the one-use-incredibly-expensive-hard-to-aim gun is ignored.


  • The "gun" has a mass at least 1,000X of the shell. Thus, 99.9% of the final velocity belongs to the shell, while only 0.1% of the final velocity belongs to the gun (which enters a decaying orbit, devastating the southern continent of your world, but that's a story for another day). Anyway, what this does is let me reasonably assume all the energy is used to push the shell.


  • Here's the whopper: we're going to assume that when they measure the explosive capability of a nuke in terms of tons of TNT, they really mean it (and the angels weep over the magnitude of this lie... but we'll get to that).



The equation of choice is F=mA. Let's be outrageous.




  • The charge is equivalent to the Tsar Bomba: 50 megatons or 210 peta-joules. So, F = 210x1015.

  • The shell is a useful 10,000 Kg. So, m = 1x104.

  • A (acceleration) = 210x1011 m/s2.


It has no resistance (space...) and there's no more force acting upon it. So our final velocity is darn close to twice the speed of light.



Which obviously can't happen



And that's why Tim's answer is correct: puff of light and heat.



OK, what's the problem?



A whomping large amount of a nuke's energy has nothing at all to do with kinetic energy. It's radiation. Oversimplifying something awful, you only have the mass of the charge to work with in terms of expanding gas (kinetic force) and a lot (if not all) of it just got converted to pure energy.



Most of the concussive force of the nuclear explosions we're familiar with are (a) the conversion of surrounding mass (air, soil, houses, people) into expanding plasma and (b) the shock created when the vacuum caused by the explosion collapses.



Which is a fancy way of saying you don't actually have that much to work with. Since you only have whatever convert-to-plasma materials are in the nuke itself (gun and shell can't be damaged by the nuke!), your gun will glow white hot1 while the equally white hot shell kinda blurps out of the muzzle and lazily meanders away.



If I got this wrong, please correct me!





Edit:



Physicist137 pointed out the flaw in my math, and I'm grateful! I had fun, but go upvote his math! I'm still happy with my conclusions, but his ~0.5c answer is a whole lot more realistic than my 2c analysis!





1And it'll probably stay white hot for decades. Space is a terrible place to dump heat.






share|improve this answer











$endgroup$









  • 1




    $begingroup$
    You got me with Tsar Bomba.
    $endgroup$
    – Mr.J
    4 hours ago










  • $begingroup$
    "So our final velocity is darn close to twice the speed of light. Which obviously can't happen" Ok so this part is fallacious for a couple of reasons, 1) because it ignores relativity, and 2) because it implies a derived speed greater than c (while ignoring relativity) counts as reducto ad absurdium. The ship is not indestructible, neither is the projectile. My question isn't about how badly the craft will blow up (it's single use, so it's already written off), but whether you can get significant velocity on the projectile that comes out of it.
    $endgroup$
    – Ingolifs
    3 hours ago










  • $begingroup$
    @Ingolifs You're not having nearly enough fun. You're missing mass between the slug and the nuke that can be converted to plasma. However, what you haven't figured out is that almost anything is more useful in space than a nuke-powered slug thrower. Especially a one-use thrower.
    $endgroup$
    – JBH
    3 hours ago






  • 2




    $begingroup$
    Hmm. Maybe I need to update the picture I provided with a mass of something ablative between nuke and projectile.
    $endgroup$
    – Ingolifs
    3 hours ago










  • $begingroup$
    I am afraid this is not correct. You are equaling a force (from newton's second law) with an energy (the bomb), and you cannot do that.
    $endgroup$
    – Physicist137
    2 hours ago


















3












$begingroup$

Tim's right, but let's have some fun explaining why.



Assumptions




  • When you said "railgun" what you meant was "slug thrower." A railgun is something specific, and you're not describing it.


  • The "gun" can't be damaged by the nuke. This is an experiment, after all.


  • Neither can the shell.


  • The practicality of the one-use-incredibly-expensive-hard-to-aim gun is ignored.


  • The "gun" has a mass at least 1,000X of the shell. Thus, 99.9% of the final velocity belongs to the shell, while only 0.1% of the final velocity belongs to the gun (which enters a decaying orbit, devastating the southern continent of your world, but that's a story for another day). Anyway, what this does is let me reasonably assume all the energy is used to push the shell.


  • Here's the whopper: we're going to assume that when they measure the explosive capability of a nuke in terms of tons of TNT, they really mean it (and the angels weep over the magnitude of this lie... but we'll get to that).



The equation of choice is F=mA. Let's be outrageous.




  • The charge is equivalent to the Tsar Bomba: 50 megatons or 210 peta-joules. So, F = 210x1015.

  • The shell is a useful 10,000 Kg. So, m = 1x104.

  • A (acceleration) = 210x1011 m/s2.


It has no resistance (space...) and there's no more force acting upon it. So our final velocity is darn close to twice the speed of light.



Which obviously can't happen



And that's why Tim's answer is correct: puff of light and heat.



OK, what's the problem?



A whomping large amount of a nuke's energy has nothing at all to do with kinetic energy. It's radiation. Oversimplifying something awful, you only have the mass of the charge to work with in terms of expanding gas (kinetic force) and a lot (if not all) of it just got converted to pure energy.



Most of the concussive force of the nuclear explosions we're familiar with are (a) the conversion of surrounding mass (air, soil, houses, people) into expanding plasma and (b) the shock created when the vacuum caused by the explosion collapses.



Which is a fancy way of saying you don't actually have that much to work with. Since you only have whatever convert-to-plasma materials are in the nuke itself (gun and shell can't be damaged by the nuke!), your gun will glow white hot1 while the equally white hot shell kinda blurps out of the muzzle and lazily meanders away.



If I got this wrong, please correct me!





Edit:



Physicist137 pointed out the flaw in my math, and I'm grateful! I had fun, but go upvote his math! I'm still happy with my conclusions, but his ~0.5c answer is a whole lot more realistic than my 2c analysis!





1And it'll probably stay white hot for decades. Space is a terrible place to dump heat.






share|improve this answer











$endgroup$









  • 1




    $begingroup$
    You got me with Tsar Bomba.
    $endgroup$
    – Mr.J
    4 hours ago










  • $begingroup$
    "So our final velocity is darn close to twice the speed of light. Which obviously can't happen" Ok so this part is fallacious for a couple of reasons, 1) because it ignores relativity, and 2) because it implies a derived speed greater than c (while ignoring relativity) counts as reducto ad absurdium. The ship is not indestructible, neither is the projectile. My question isn't about how badly the craft will blow up (it's single use, so it's already written off), but whether you can get significant velocity on the projectile that comes out of it.
    $endgroup$
    – Ingolifs
    3 hours ago










  • $begingroup$
    @Ingolifs You're not having nearly enough fun. You're missing mass between the slug and the nuke that can be converted to plasma. However, what you haven't figured out is that almost anything is more useful in space than a nuke-powered slug thrower. Especially a one-use thrower.
    $endgroup$
    – JBH
    3 hours ago






  • 2




    $begingroup$
    Hmm. Maybe I need to update the picture I provided with a mass of something ablative between nuke and projectile.
    $endgroup$
    – Ingolifs
    3 hours ago










  • $begingroup$
    I am afraid this is not correct. You are equaling a force (from newton's second law) with an energy (the bomb), and you cannot do that.
    $endgroup$
    – Physicist137
    2 hours ago
















3












3








3





$begingroup$

Tim's right, but let's have some fun explaining why.



Assumptions




  • When you said "railgun" what you meant was "slug thrower." A railgun is something specific, and you're not describing it.


  • The "gun" can't be damaged by the nuke. This is an experiment, after all.


  • Neither can the shell.


  • The practicality of the one-use-incredibly-expensive-hard-to-aim gun is ignored.


  • The "gun" has a mass at least 1,000X of the shell. Thus, 99.9% of the final velocity belongs to the shell, while only 0.1% of the final velocity belongs to the gun (which enters a decaying orbit, devastating the southern continent of your world, but that's a story for another day). Anyway, what this does is let me reasonably assume all the energy is used to push the shell.


  • Here's the whopper: we're going to assume that when they measure the explosive capability of a nuke in terms of tons of TNT, they really mean it (and the angels weep over the magnitude of this lie... but we'll get to that).



The equation of choice is F=mA. Let's be outrageous.




  • The charge is equivalent to the Tsar Bomba: 50 megatons or 210 peta-joules. So, F = 210x1015.

  • The shell is a useful 10,000 Kg. So, m = 1x104.

  • A (acceleration) = 210x1011 m/s2.


It has no resistance (space...) and there's no more force acting upon it. So our final velocity is darn close to twice the speed of light.



Which obviously can't happen



And that's why Tim's answer is correct: puff of light and heat.



OK, what's the problem?



A whomping large amount of a nuke's energy has nothing at all to do with kinetic energy. It's radiation. Oversimplifying something awful, you only have the mass of the charge to work with in terms of expanding gas (kinetic force) and a lot (if not all) of it just got converted to pure energy.



Most of the concussive force of the nuclear explosions we're familiar with are (a) the conversion of surrounding mass (air, soil, houses, people) into expanding plasma and (b) the shock created when the vacuum caused by the explosion collapses.



Which is a fancy way of saying you don't actually have that much to work with. Since you only have whatever convert-to-plasma materials are in the nuke itself (gun and shell can't be damaged by the nuke!), your gun will glow white hot1 while the equally white hot shell kinda blurps out of the muzzle and lazily meanders away.



If I got this wrong, please correct me!





Edit:



Physicist137 pointed out the flaw in my math, and I'm grateful! I had fun, but go upvote his math! I'm still happy with my conclusions, but his ~0.5c answer is a whole lot more realistic than my 2c analysis!





1And it'll probably stay white hot for decades. Space is a terrible place to dump heat.






share|improve this answer











$endgroup$



Tim's right, but let's have some fun explaining why.



Assumptions




  • When you said "railgun" what you meant was "slug thrower." A railgun is something specific, and you're not describing it.


  • The "gun" can't be damaged by the nuke. This is an experiment, after all.


  • Neither can the shell.


  • The practicality of the one-use-incredibly-expensive-hard-to-aim gun is ignored.


  • The "gun" has a mass at least 1,000X of the shell. Thus, 99.9% of the final velocity belongs to the shell, while only 0.1% of the final velocity belongs to the gun (which enters a decaying orbit, devastating the southern continent of your world, but that's a story for another day). Anyway, what this does is let me reasonably assume all the energy is used to push the shell.


  • Here's the whopper: we're going to assume that when they measure the explosive capability of a nuke in terms of tons of TNT, they really mean it (and the angels weep over the magnitude of this lie... but we'll get to that).



The equation of choice is F=mA. Let's be outrageous.




  • The charge is equivalent to the Tsar Bomba: 50 megatons or 210 peta-joules. So, F = 210x1015.

  • The shell is a useful 10,000 Kg. So, m = 1x104.

  • A (acceleration) = 210x1011 m/s2.


It has no resistance (space...) and there's no more force acting upon it. So our final velocity is darn close to twice the speed of light.



Which obviously can't happen



And that's why Tim's answer is correct: puff of light and heat.



OK, what's the problem?



A whomping large amount of a nuke's energy has nothing at all to do with kinetic energy. It's radiation. Oversimplifying something awful, you only have the mass of the charge to work with in terms of expanding gas (kinetic force) and a lot (if not all) of it just got converted to pure energy.



Most of the concussive force of the nuclear explosions we're familiar with are (a) the conversion of surrounding mass (air, soil, houses, people) into expanding plasma and (b) the shock created when the vacuum caused by the explosion collapses.



Which is a fancy way of saying you don't actually have that much to work with. Since you only have whatever convert-to-plasma materials are in the nuke itself (gun and shell can't be damaged by the nuke!), your gun will glow white hot1 while the equally white hot shell kinda blurps out of the muzzle and lazily meanders away.



If I got this wrong, please correct me!





Edit:



Physicist137 pointed out the flaw in my math, and I'm grateful! I had fun, but go upvote his math! I'm still happy with my conclusions, but his ~0.5c answer is a whole lot more realistic than my 2c analysis!





1And it'll probably stay white hot for decades. Space is a terrible place to dump heat.







share|improve this answer














share|improve this answer



share|improve this answer








edited 1 hour ago

























answered 4 hours ago









JBHJBH

41.4k591197




41.4k591197








  • 1




    $begingroup$
    You got me with Tsar Bomba.
    $endgroup$
    – Mr.J
    4 hours ago










  • $begingroup$
    "So our final velocity is darn close to twice the speed of light. Which obviously can't happen" Ok so this part is fallacious for a couple of reasons, 1) because it ignores relativity, and 2) because it implies a derived speed greater than c (while ignoring relativity) counts as reducto ad absurdium. The ship is not indestructible, neither is the projectile. My question isn't about how badly the craft will blow up (it's single use, so it's already written off), but whether you can get significant velocity on the projectile that comes out of it.
    $endgroup$
    – Ingolifs
    3 hours ago










  • $begingroup$
    @Ingolifs You're not having nearly enough fun. You're missing mass between the slug and the nuke that can be converted to plasma. However, what you haven't figured out is that almost anything is more useful in space than a nuke-powered slug thrower. Especially a one-use thrower.
    $endgroup$
    – JBH
    3 hours ago






  • 2




    $begingroup$
    Hmm. Maybe I need to update the picture I provided with a mass of something ablative between nuke and projectile.
    $endgroup$
    – Ingolifs
    3 hours ago










  • $begingroup$
    I am afraid this is not correct. You are equaling a force (from newton's second law) with an energy (the bomb), and you cannot do that.
    $endgroup$
    – Physicist137
    2 hours ago
















  • 1




    $begingroup$
    You got me with Tsar Bomba.
    $endgroup$
    – Mr.J
    4 hours ago










  • $begingroup$
    "So our final velocity is darn close to twice the speed of light. Which obviously can't happen" Ok so this part is fallacious for a couple of reasons, 1) because it ignores relativity, and 2) because it implies a derived speed greater than c (while ignoring relativity) counts as reducto ad absurdium. The ship is not indestructible, neither is the projectile. My question isn't about how badly the craft will blow up (it's single use, so it's already written off), but whether you can get significant velocity on the projectile that comes out of it.
    $endgroup$
    – Ingolifs
    3 hours ago










  • $begingroup$
    @Ingolifs You're not having nearly enough fun. You're missing mass between the slug and the nuke that can be converted to plasma. However, what you haven't figured out is that almost anything is more useful in space than a nuke-powered slug thrower. Especially a one-use thrower.
    $endgroup$
    – JBH
    3 hours ago






  • 2




    $begingroup$
    Hmm. Maybe I need to update the picture I provided with a mass of something ablative between nuke and projectile.
    $endgroup$
    – Ingolifs
    3 hours ago










  • $begingroup$
    I am afraid this is not correct. You are equaling a force (from newton's second law) with an energy (the bomb), and you cannot do that.
    $endgroup$
    – Physicist137
    2 hours ago










1




1




$begingroup$
You got me with Tsar Bomba.
$endgroup$
– Mr.J
4 hours ago




$begingroup$
You got me with Tsar Bomba.
$endgroup$
– Mr.J
4 hours ago












$begingroup$
"So our final velocity is darn close to twice the speed of light. Which obviously can't happen" Ok so this part is fallacious for a couple of reasons, 1) because it ignores relativity, and 2) because it implies a derived speed greater than c (while ignoring relativity) counts as reducto ad absurdium. The ship is not indestructible, neither is the projectile. My question isn't about how badly the craft will blow up (it's single use, so it's already written off), but whether you can get significant velocity on the projectile that comes out of it.
$endgroup$
– Ingolifs
3 hours ago




$begingroup$
"So our final velocity is darn close to twice the speed of light. Which obviously can't happen" Ok so this part is fallacious for a couple of reasons, 1) because it ignores relativity, and 2) because it implies a derived speed greater than c (while ignoring relativity) counts as reducto ad absurdium. The ship is not indestructible, neither is the projectile. My question isn't about how badly the craft will blow up (it's single use, so it's already written off), but whether you can get significant velocity on the projectile that comes out of it.
$endgroup$
– Ingolifs
3 hours ago












$begingroup$
@Ingolifs You're not having nearly enough fun. You're missing mass between the slug and the nuke that can be converted to plasma. However, what you haven't figured out is that almost anything is more useful in space than a nuke-powered slug thrower. Especially a one-use thrower.
$endgroup$
– JBH
3 hours ago




$begingroup$
@Ingolifs You're not having nearly enough fun. You're missing mass between the slug and the nuke that can be converted to plasma. However, what you haven't figured out is that almost anything is more useful in space than a nuke-powered slug thrower. Especially a one-use thrower.
$endgroup$
– JBH
3 hours ago




2




2




$begingroup$
Hmm. Maybe I need to update the picture I provided with a mass of something ablative between nuke and projectile.
$endgroup$
– Ingolifs
3 hours ago




$begingroup$
Hmm. Maybe I need to update the picture I provided with a mass of something ablative between nuke and projectile.
$endgroup$
– Ingolifs
3 hours ago












$begingroup$
I am afraid this is not correct. You are equaling a force (from newton's second law) with an energy (the bomb), and you cannot do that.
$endgroup$
– Physicist137
2 hours ago






$begingroup$
I am afraid this is not correct. You are equaling a force (from newton's second law) with an energy (the bomb), and you cannot do that.
$endgroup$
– Physicist137
2 hours ago













1












$begingroup$

There's a simple problem with this: shock and vibe.



Your projectile has to travel a very long distance before hitting. Your specification of "around the limit of detection by IR" is sufficient to be quite certain you will never hit your enemy with a dumb slug. You simply can't aim the gun that accurately. This is doubly true for the fact that you're about to set off a nuclear explosion (probably well more than doubly true).



This means you're certainly using a smart weapon, with onboard propellants, sensors, and computers, much like we see in all of our own ballistic defense programs on planet Earth.



So here's the real issue. Launching things fast (space fast) is hard. You have to accelerate rather quickly. Shock and vibe issues during launch are astonishingly difficult to deal with. The harder you accelerate, the nastier the requirements. Navy rail guns face serious challenges in hardening smart rounds so that it survives being fired (tests are typically done with dumb Tungsten slugs today, but it is recognized that they will need to be guided eventually to strike distant targets). You have invented a new level of "difficult launch conditions." Railguns can maintain a maximized shock and vibe for the length of the rail far better than a simple nuclear gun can.



Launch Conditions: Must survive being at ground zero of a 25MT thermonuclear bomb detonation, in a confined space designed to maximize the impact of the thermonuclear bomb on the projectile.



Ouch!



What you really want to do is stick to railguns. If you design a projectile that can survive the forces of a nuclear bomb, you immediately put that same projectile into a railgun, and get more performance out of it (railguns can accelerate linearly along the entire length of the rails, while this gun peters out as the pressure wave weakens)






share|improve this answer









$endgroup$













  • $begingroup$
    Good answer. There is a 'combat regime' I guess you'd call it, where you can see your enemy but do not have the ability to hit them, because the time-of-flight of your projectiles is longer than the time needed to get out of the way. If you make your projectile faster, you have a better chance of hitting, providing you do not sacrifice accuracy. Your point about projectiles needing guidance in any case is a good one. That will always limit the acceleration of the projectile to what the guidance computer system can survive. In which case, using the nukes for Orion drive would be better.
    $endgroup$
    – Ingolifs
    1 hour ago










  • $begingroup$
    If I remember my rules for intercepts correctly, faster doesn't actually make it much better. Reaction times are important, and faster will make it better with respect to reactions, but once you actually get to the demanding task of moving your ship out of the way of the round, faster no lnoger helps. They have less time to move, but you have less time to respond to their movement, so it cancels out.
    $endgroup$
    – Cort Ammon
    1 hour ago
















1












$begingroup$

There's a simple problem with this: shock and vibe.



Your projectile has to travel a very long distance before hitting. Your specification of "around the limit of detection by IR" is sufficient to be quite certain you will never hit your enemy with a dumb slug. You simply can't aim the gun that accurately. This is doubly true for the fact that you're about to set off a nuclear explosion (probably well more than doubly true).



This means you're certainly using a smart weapon, with onboard propellants, sensors, and computers, much like we see in all of our own ballistic defense programs on planet Earth.



So here's the real issue. Launching things fast (space fast) is hard. You have to accelerate rather quickly. Shock and vibe issues during launch are astonishingly difficult to deal with. The harder you accelerate, the nastier the requirements. Navy rail guns face serious challenges in hardening smart rounds so that it survives being fired (tests are typically done with dumb Tungsten slugs today, but it is recognized that they will need to be guided eventually to strike distant targets). You have invented a new level of "difficult launch conditions." Railguns can maintain a maximized shock and vibe for the length of the rail far better than a simple nuclear gun can.



Launch Conditions: Must survive being at ground zero of a 25MT thermonuclear bomb detonation, in a confined space designed to maximize the impact of the thermonuclear bomb on the projectile.



Ouch!



What you really want to do is stick to railguns. If you design a projectile that can survive the forces of a nuclear bomb, you immediately put that same projectile into a railgun, and get more performance out of it (railguns can accelerate linearly along the entire length of the rails, while this gun peters out as the pressure wave weakens)






share|improve this answer









$endgroup$













  • $begingroup$
    Good answer. There is a 'combat regime' I guess you'd call it, where you can see your enemy but do not have the ability to hit them, because the time-of-flight of your projectiles is longer than the time needed to get out of the way. If you make your projectile faster, you have a better chance of hitting, providing you do not sacrifice accuracy. Your point about projectiles needing guidance in any case is a good one. That will always limit the acceleration of the projectile to what the guidance computer system can survive. In which case, using the nukes for Orion drive would be better.
    $endgroup$
    – Ingolifs
    1 hour ago










  • $begingroup$
    If I remember my rules for intercepts correctly, faster doesn't actually make it much better. Reaction times are important, and faster will make it better with respect to reactions, but once you actually get to the demanding task of moving your ship out of the way of the round, faster no lnoger helps. They have less time to move, but you have less time to respond to their movement, so it cancels out.
    $endgroup$
    – Cort Ammon
    1 hour ago














1












1








1





$begingroup$

There's a simple problem with this: shock and vibe.



Your projectile has to travel a very long distance before hitting. Your specification of "around the limit of detection by IR" is sufficient to be quite certain you will never hit your enemy with a dumb slug. You simply can't aim the gun that accurately. This is doubly true for the fact that you're about to set off a nuclear explosion (probably well more than doubly true).



This means you're certainly using a smart weapon, with onboard propellants, sensors, and computers, much like we see in all of our own ballistic defense programs on planet Earth.



So here's the real issue. Launching things fast (space fast) is hard. You have to accelerate rather quickly. Shock and vibe issues during launch are astonishingly difficult to deal with. The harder you accelerate, the nastier the requirements. Navy rail guns face serious challenges in hardening smart rounds so that it survives being fired (tests are typically done with dumb Tungsten slugs today, but it is recognized that they will need to be guided eventually to strike distant targets). You have invented a new level of "difficult launch conditions." Railguns can maintain a maximized shock and vibe for the length of the rail far better than a simple nuclear gun can.



Launch Conditions: Must survive being at ground zero of a 25MT thermonuclear bomb detonation, in a confined space designed to maximize the impact of the thermonuclear bomb on the projectile.



Ouch!



What you really want to do is stick to railguns. If you design a projectile that can survive the forces of a nuclear bomb, you immediately put that same projectile into a railgun, and get more performance out of it (railguns can accelerate linearly along the entire length of the rails, while this gun peters out as the pressure wave weakens)






share|improve this answer









$endgroup$



There's a simple problem with this: shock and vibe.



Your projectile has to travel a very long distance before hitting. Your specification of "around the limit of detection by IR" is sufficient to be quite certain you will never hit your enemy with a dumb slug. You simply can't aim the gun that accurately. This is doubly true for the fact that you're about to set off a nuclear explosion (probably well more than doubly true).



This means you're certainly using a smart weapon, with onboard propellants, sensors, and computers, much like we see in all of our own ballistic defense programs on planet Earth.



So here's the real issue. Launching things fast (space fast) is hard. You have to accelerate rather quickly. Shock and vibe issues during launch are astonishingly difficult to deal with. The harder you accelerate, the nastier the requirements. Navy rail guns face serious challenges in hardening smart rounds so that it survives being fired (tests are typically done with dumb Tungsten slugs today, but it is recognized that they will need to be guided eventually to strike distant targets). You have invented a new level of "difficult launch conditions." Railguns can maintain a maximized shock and vibe for the length of the rail far better than a simple nuclear gun can.



Launch Conditions: Must survive being at ground zero of a 25MT thermonuclear bomb detonation, in a confined space designed to maximize the impact of the thermonuclear bomb on the projectile.



Ouch!



What you really want to do is stick to railguns. If you design a projectile that can survive the forces of a nuclear bomb, you immediately put that same projectile into a railgun, and get more performance out of it (railguns can accelerate linearly along the entire length of the rails, while this gun peters out as the pressure wave weakens)







share|improve this answer












share|improve this answer



share|improve this answer










answered 1 hour ago









Cort AmmonCort Ammon

109k17187385




109k17187385












  • $begingroup$
    Good answer. There is a 'combat regime' I guess you'd call it, where you can see your enemy but do not have the ability to hit them, because the time-of-flight of your projectiles is longer than the time needed to get out of the way. If you make your projectile faster, you have a better chance of hitting, providing you do not sacrifice accuracy. Your point about projectiles needing guidance in any case is a good one. That will always limit the acceleration of the projectile to what the guidance computer system can survive. In which case, using the nukes for Orion drive would be better.
    $endgroup$
    – Ingolifs
    1 hour ago










  • $begingroup$
    If I remember my rules for intercepts correctly, faster doesn't actually make it much better. Reaction times are important, and faster will make it better with respect to reactions, but once you actually get to the demanding task of moving your ship out of the way of the round, faster no lnoger helps. They have less time to move, but you have less time to respond to their movement, so it cancels out.
    $endgroup$
    – Cort Ammon
    1 hour ago


















  • $begingroup$
    Good answer. There is a 'combat regime' I guess you'd call it, where you can see your enemy but do not have the ability to hit them, because the time-of-flight of your projectiles is longer than the time needed to get out of the way. If you make your projectile faster, you have a better chance of hitting, providing you do not sacrifice accuracy. Your point about projectiles needing guidance in any case is a good one. That will always limit the acceleration of the projectile to what the guidance computer system can survive. In which case, using the nukes for Orion drive would be better.
    $endgroup$
    – Ingolifs
    1 hour ago










  • $begingroup$
    If I remember my rules for intercepts correctly, faster doesn't actually make it much better. Reaction times are important, and faster will make it better with respect to reactions, but once you actually get to the demanding task of moving your ship out of the way of the round, faster no lnoger helps. They have less time to move, but you have less time to respond to their movement, so it cancels out.
    $endgroup$
    – Cort Ammon
    1 hour ago
















$begingroup$
Good answer. There is a 'combat regime' I guess you'd call it, where you can see your enemy but do not have the ability to hit them, because the time-of-flight of your projectiles is longer than the time needed to get out of the way. If you make your projectile faster, you have a better chance of hitting, providing you do not sacrifice accuracy. Your point about projectiles needing guidance in any case is a good one. That will always limit the acceleration of the projectile to what the guidance computer system can survive. In which case, using the nukes for Orion drive would be better.
$endgroup$
– Ingolifs
1 hour ago




$begingroup$
Good answer. There is a 'combat regime' I guess you'd call it, where you can see your enemy but do not have the ability to hit them, because the time-of-flight of your projectiles is longer than the time needed to get out of the way. If you make your projectile faster, you have a better chance of hitting, providing you do not sacrifice accuracy. Your point about projectiles needing guidance in any case is a good one. That will always limit the acceleration of the projectile to what the guidance computer system can survive. In which case, using the nukes for Orion drive would be better.
$endgroup$
– Ingolifs
1 hour ago












$begingroup$
If I remember my rules for intercepts correctly, faster doesn't actually make it much better. Reaction times are important, and faster will make it better with respect to reactions, but once you actually get to the demanding task of moving your ship out of the way of the round, faster no lnoger helps. They have less time to move, but you have less time to respond to their movement, so it cancels out.
$endgroup$
– Cort Ammon
1 hour ago




$begingroup$
If I remember my rules for intercepts correctly, faster doesn't actually make it much better. Reaction times are important, and faster will make it better with respect to reactions, but once you actually get to the demanding task of moving your ship out of the way of the round, faster no lnoger helps. They have less time to move, but you have less time to respond to their movement, so it cancels out.
$endgroup$
– Cort Ammon
1 hour ago











-1












$begingroup$

Your single use ship will disappear in a puff of light and heat.



The problem with what you're describing is that a nuclear bomb is exactly the wrong type of energy source for a rail gun as it generates thermal heat and radiation, not kinetic energy, in space.



We've all read about the blast waves off nuclear explosions and how powerful they are; there is even some research into this that shows that blast (kinetic) energy comprises roughly half the energy released in a nuclear explosion. The trouble is, that blast energy is essentially the atmosphere super heating from the thermal energy released, and therefore rapidly increasing in pressure in an unconfined space.



Nuclear bombs are not designed to release kinetic energy; they're designed to release thermal energy. This can still work because of a technique known as Ablative propulsion, which amounts to superheating a surface of an object to a point where the surface becomes a plasma that pushes the object rapidly in the opposite direction to that in which the heated surface is facing.



This kind of thinking about propulsion seems to be behind Nuclear Pulse Propulsion theoretical engine systems like that originally planned for Project Orion. My reading on this to date seems to indicate that the plan was to detonate nuclear bombs outside the ship and have the blast hit a pusher plate on the back of the ship, pushing it in that direction. What is unclear in all those mainstream articles is how that blast wave is generated but the best I can come up with is that the mass of the bomb itself would become a superheated plasma that would push against the rear plate of the ship. The closest I could come up with in terms of reference to how nuclear energy can be converted to kinetic energy in space was a reference to Electrical v Direct Thrust on fusion drives, but even it talks about the plasma having to be contained to sustain the reaction which to my mind seems contradictory.



If this isn't the thinking behind nuclear pulse theory and you know what is, leave it in comments and I'll integrate it into my answer with full credits. But back to the matter at hand...



In space, there's no atmosphere to heat up, so no localised pressure differential, therefore no blast wave. What WILL happen is that the heat energy will essentially vapourise your ship and the projectile through heating them all equally.



To be fair, this will result in the aforementioned ablative propulsion for your rail projectile; as the weak point in the containment, it's going to break off (or out) first but without running some serious math I have no idea just how quickly it will escape the barrel, how much of it will be left to hit the target, or even if it would escape the barrel at all before the thermal energy expands it (and contracts the barrel) before it escapes.



Before anyone says it, yes, the barrel will constrict. As the metal expands, the inner metal of the barrel also expands, meaning that the hole gets smaller through swelling.



Guns like cannon, pistols, rifles and rail guns work through the kinetic energy of expanding gases. The idea is that gunpowder, when burnt, releases a massive amount of hot gas into a confined area, forcing out the weakest point of the containment chamber, which just happens to be the bullet. This projects it suddenly in the direction of your choosing. Yes, burnt gunpowder is hot, but it's not the heat that shoots out the bullet; it's the expanding gases.



In space, your setup has no atmosphere around it to expand, and even if it did, the kinetic energy transferred to it only represents about half the energy that a nuclear device can produce, which makes it an inefficient energy source for what you're trying to do, and would require such a sturdy, heavy ship to withstand the vapourisation long enough to transfer the bulk of the energy to the projectile so as to make it uneconomic as an investment.



For my money, you'd be far better off investing in a nuclear powerplant that feeds electrical capacitors that are capable of releasing a sudden energy burst of electricity to propel your bullet. Far more efficient process of converting thermal to kinetic energy (via temporarily being electricity), reusable and less dangerous to the rest of your fleet by not creating debris fields with every shot.






share|improve this answer











$endgroup$













  • $begingroup$
    While I too question its practicality, is this not roughly analogous to the nuclear pulse rocket concept?
    $endgroup$
    – Cort Ammon
    6 hours ago










  • $begingroup$
    @CortAmmon you're right; I'd forgotten about that until you mentioned it. I just did a quick check on it and most of the material I can find on this talks about specific impulses, but doesn't actually go into detail about how the blast wave in space actually works. It seems to be related to the surrounding mass being converted to plasma. I'll do some more research and edit accordingly within the next couple of hours.
    $endgroup$
    – Tim B II
    6 hours ago










  • $begingroup$
    I'm not convinced by this answer. Operation Plumbbob has shown that such a thing is at least possible, though it may require a separation of gas between the nuclear explosive and the projectile, and probably a much weaker nuke than the likes of Fat Man (let alone Tsar Bomba).
    $endgroup$
    – Ingolifs
    3 hours ago










  • $begingroup$
    @Ingolifs the trouble with comparing Operation Plumbbob to this scenario is that it was conducted on Earth, not space. Earth has an atmosphere that can be heated as an expanding gas and plasma, space doesn't have a medium in which to propagate the blast wave.
    $endgroup$
    – Tim B II
    2 hours ago










  • $begingroup$
    I've edited my original question to include a mass of material between the projectile and the nuclear explosive, that vaporises into gas and plasma.
    $endgroup$
    – Ingolifs
    2 hours ago
















-1












$begingroup$

Your single use ship will disappear in a puff of light and heat.



The problem with what you're describing is that a nuclear bomb is exactly the wrong type of energy source for a rail gun as it generates thermal heat and radiation, not kinetic energy, in space.



We've all read about the blast waves off nuclear explosions and how powerful they are; there is even some research into this that shows that blast (kinetic) energy comprises roughly half the energy released in a nuclear explosion. The trouble is, that blast energy is essentially the atmosphere super heating from the thermal energy released, and therefore rapidly increasing in pressure in an unconfined space.



Nuclear bombs are not designed to release kinetic energy; they're designed to release thermal energy. This can still work because of a technique known as Ablative propulsion, which amounts to superheating a surface of an object to a point where the surface becomes a plasma that pushes the object rapidly in the opposite direction to that in which the heated surface is facing.



This kind of thinking about propulsion seems to be behind Nuclear Pulse Propulsion theoretical engine systems like that originally planned for Project Orion. My reading on this to date seems to indicate that the plan was to detonate nuclear bombs outside the ship and have the blast hit a pusher plate on the back of the ship, pushing it in that direction. What is unclear in all those mainstream articles is how that blast wave is generated but the best I can come up with is that the mass of the bomb itself would become a superheated plasma that would push against the rear plate of the ship. The closest I could come up with in terms of reference to how nuclear energy can be converted to kinetic energy in space was a reference to Electrical v Direct Thrust on fusion drives, but even it talks about the plasma having to be contained to sustain the reaction which to my mind seems contradictory.



If this isn't the thinking behind nuclear pulse theory and you know what is, leave it in comments and I'll integrate it into my answer with full credits. But back to the matter at hand...



In space, there's no atmosphere to heat up, so no localised pressure differential, therefore no blast wave. What WILL happen is that the heat energy will essentially vapourise your ship and the projectile through heating them all equally.



To be fair, this will result in the aforementioned ablative propulsion for your rail projectile; as the weak point in the containment, it's going to break off (or out) first but without running some serious math I have no idea just how quickly it will escape the barrel, how much of it will be left to hit the target, or even if it would escape the barrel at all before the thermal energy expands it (and contracts the barrel) before it escapes.



Before anyone says it, yes, the barrel will constrict. As the metal expands, the inner metal of the barrel also expands, meaning that the hole gets smaller through swelling.



Guns like cannon, pistols, rifles and rail guns work through the kinetic energy of expanding gases. The idea is that gunpowder, when burnt, releases a massive amount of hot gas into a confined area, forcing out the weakest point of the containment chamber, which just happens to be the bullet. This projects it suddenly in the direction of your choosing. Yes, burnt gunpowder is hot, but it's not the heat that shoots out the bullet; it's the expanding gases.



In space, your setup has no atmosphere around it to expand, and even if it did, the kinetic energy transferred to it only represents about half the energy that a nuclear device can produce, which makes it an inefficient energy source for what you're trying to do, and would require such a sturdy, heavy ship to withstand the vapourisation long enough to transfer the bulk of the energy to the projectile so as to make it uneconomic as an investment.



For my money, you'd be far better off investing in a nuclear powerplant that feeds electrical capacitors that are capable of releasing a sudden energy burst of electricity to propel your bullet. Far more efficient process of converting thermal to kinetic energy (via temporarily being electricity), reusable and less dangerous to the rest of your fleet by not creating debris fields with every shot.






share|improve this answer











$endgroup$













  • $begingroup$
    While I too question its practicality, is this not roughly analogous to the nuclear pulse rocket concept?
    $endgroup$
    – Cort Ammon
    6 hours ago










  • $begingroup$
    @CortAmmon you're right; I'd forgotten about that until you mentioned it. I just did a quick check on it and most of the material I can find on this talks about specific impulses, but doesn't actually go into detail about how the blast wave in space actually works. It seems to be related to the surrounding mass being converted to plasma. I'll do some more research and edit accordingly within the next couple of hours.
    $endgroup$
    – Tim B II
    6 hours ago










  • $begingroup$
    I'm not convinced by this answer. Operation Plumbbob has shown that such a thing is at least possible, though it may require a separation of gas between the nuclear explosive and the projectile, and probably a much weaker nuke than the likes of Fat Man (let alone Tsar Bomba).
    $endgroup$
    – Ingolifs
    3 hours ago










  • $begingroup$
    @Ingolifs the trouble with comparing Operation Plumbbob to this scenario is that it was conducted on Earth, not space. Earth has an atmosphere that can be heated as an expanding gas and plasma, space doesn't have a medium in which to propagate the blast wave.
    $endgroup$
    – Tim B II
    2 hours ago










  • $begingroup$
    I've edited my original question to include a mass of material between the projectile and the nuclear explosive, that vaporises into gas and plasma.
    $endgroup$
    – Ingolifs
    2 hours ago














-1












-1








-1





$begingroup$

Your single use ship will disappear in a puff of light and heat.



The problem with what you're describing is that a nuclear bomb is exactly the wrong type of energy source for a rail gun as it generates thermal heat and radiation, not kinetic energy, in space.



We've all read about the blast waves off nuclear explosions and how powerful they are; there is even some research into this that shows that blast (kinetic) energy comprises roughly half the energy released in a nuclear explosion. The trouble is, that blast energy is essentially the atmosphere super heating from the thermal energy released, and therefore rapidly increasing in pressure in an unconfined space.



Nuclear bombs are not designed to release kinetic energy; they're designed to release thermal energy. This can still work because of a technique known as Ablative propulsion, which amounts to superheating a surface of an object to a point where the surface becomes a plasma that pushes the object rapidly in the opposite direction to that in which the heated surface is facing.



This kind of thinking about propulsion seems to be behind Nuclear Pulse Propulsion theoretical engine systems like that originally planned for Project Orion. My reading on this to date seems to indicate that the plan was to detonate nuclear bombs outside the ship and have the blast hit a pusher plate on the back of the ship, pushing it in that direction. What is unclear in all those mainstream articles is how that blast wave is generated but the best I can come up with is that the mass of the bomb itself would become a superheated plasma that would push against the rear plate of the ship. The closest I could come up with in terms of reference to how nuclear energy can be converted to kinetic energy in space was a reference to Electrical v Direct Thrust on fusion drives, but even it talks about the plasma having to be contained to sustain the reaction which to my mind seems contradictory.



If this isn't the thinking behind nuclear pulse theory and you know what is, leave it in comments and I'll integrate it into my answer with full credits. But back to the matter at hand...



In space, there's no atmosphere to heat up, so no localised pressure differential, therefore no blast wave. What WILL happen is that the heat energy will essentially vapourise your ship and the projectile through heating them all equally.



To be fair, this will result in the aforementioned ablative propulsion for your rail projectile; as the weak point in the containment, it's going to break off (or out) first but without running some serious math I have no idea just how quickly it will escape the barrel, how much of it will be left to hit the target, or even if it would escape the barrel at all before the thermal energy expands it (and contracts the barrel) before it escapes.



Before anyone says it, yes, the barrel will constrict. As the metal expands, the inner metal of the barrel also expands, meaning that the hole gets smaller through swelling.



Guns like cannon, pistols, rifles and rail guns work through the kinetic energy of expanding gases. The idea is that gunpowder, when burnt, releases a massive amount of hot gas into a confined area, forcing out the weakest point of the containment chamber, which just happens to be the bullet. This projects it suddenly in the direction of your choosing. Yes, burnt gunpowder is hot, but it's not the heat that shoots out the bullet; it's the expanding gases.



In space, your setup has no atmosphere around it to expand, and even if it did, the kinetic energy transferred to it only represents about half the energy that a nuclear device can produce, which makes it an inefficient energy source for what you're trying to do, and would require such a sturdy, heavy ship to withstand the vapourisation long enough to transfer the bulk of the energy to the projectile so as to make it uneconomic as an investment.



For my money, you'd be far better off investing in a nuclear powerplant that feeds electrical capacitors that are capable of releasing a sudden energy burst of electricity to propel your bullet. Far more efficient process of converting thermal to kinetic energy (via temporarily being electricity), reusable and less dangerous to the rest of your fleet by not creating debris fields with every shot.






share|improve this answer











$endgroup$



Your single use ship will disappear in a puff of light and heat.



The problem with what you're describing is that a nuclear bomb is exactly the wrong type of energy source for a rail gun as it generates thermal heat and radiation, not kinetic energy, in space.



We've all read about the blast waves off nuclear explosions and how powerful they are; there is even some research into this that shows that blast (kinetic) energy comprises roughly half the energy released in a nuclear explosion. The trouble is, that blast energy is essentially the atmosphere super heating from the thermal energy released, and therefore rapidly increasing in pressure in an unconfined space.



Nuclear bombs are not designed to release kinetic energy; they're designed to release thermal energy. This can still work because of a technique known as Ablative propulsion, which amounts to superheating a surface of an object to a point where the surface becomes a plasma that pushes the object rapidly in the opposite direction to that in which the heated surface is facing.



This kind of thinking about propulsion seems to be behind Nuclear Pulse Propulsion theoretical engine systems like that originally planned for Project Orion. My reading on this to date seems to indicate that the plan was to detonate nuclear bombs outside the ship and have the blast hit a pusher plate on the back of the ship, pushing it in that direction. What is unclear in all those mainstream articles is how that blast wave is generated but the best I can come up with is that the mass of the bomb itself would become a superheated plasma that would push against the rear plate of the ship. The closest I could come up with in terms of reference to how nuclear energy can be converted to kinetic energy in space was a reference to Electrical v Direct Thrust on fusion drives, but even it talks about the plasma having to be contained to sustain the reaction which to my mind seems contradictory.



If this isn't the thinking behind nuclear pulse theory and you know what is, leave it in comments and I'll integrate it into my answer with full credits. But back to the matter at hand...



In space, there's no atmosphere to heat up, so no localised pressure differential, therefore no blast wave. What WILL happen is that the heat energy will essentially vapourise your ship and the projectile through heating them all equally.



To be fair, this will result in the aforementioned ablative propulsion for your rail projectile; as the weak point in the containment, it's going to break off (or out) first but without running some serious math I have no idea just how quickly it will escape the barrel, how much of it will be left to hit the target, or even if it would escape the barrel at all before the thermal energy expands it (and contracts the barrel) before it escapes.



Before anyone says it, yes, the barrel will constrict. As the metal expands, the inner metal of the barrel also expands, meaning that the hole gets smaller through swelling.



Guns like cannon, pistols, rifles and rail guns work through the kinetic energy of expanding gases. The idea is that gunpowder, when burnt, releases a massive amount of hot gas into a confined area, forcing out the weakest point of the containment chamber, which just happens to be the bullet. This projects it suddenly in the direction of your choosing. Yes, burnt gunpowder is hot, but it's not the heat that shoots out the bullet; it's the expanding gases.



In space, your setup has no atmosphere around it to expand, and even if it did, the kinetic energy transferred to it only represents about half the energy that a nuclear device can produce, which makes it an inefficient energy source for what you're trying to do, and would require such a sturdy, heavy ship to withstand the vapourisation long enough to transfer the bulk of the energy to the projectile so as to make it uneconomic as an investment.



For my money, you'd be far better off investing in a nuclear powerplant that feeds electrical capacitors that are capable of releasing a sudden energy burst of electricity to propel your bullet. Far more efficient process of converting thermal to kinetic energy (via temporarily being electricity), reusable and less dangerous to the rest of your fleet by not creating debris fields with every shot.







share|improve this answer














share|improve this answer



share|improve this answer








edited 5 hours ago

























answered 6 hours ago









Tim B IITim B II

26.7k659112




26.7k659112












  • $begingroup$
    While I too question its practicality, is this not roughly analogous to the nuclear pulse rocket concept?
    $endgroup$
    – Cort Ammon
    6 hours ago










  • $begingroup$
    @CortAmmon you're right; I'd forgotten about that until you mentioned it. I just did a quick check on it and most of the material I can find on this talks about specific impulses, but doesn't actually go into detail about how the blast wave in space actually works. It seems to be related to the surrounding mass being converted to plasma. I'll do some more research and edit accordingly within the next couple of hours.
    $endgroup$
    – Tim B II
    6 hours ago










  • $begingroup$
    I'm not convinced by this answer. Operation Plumbbob has shown that such a thing is at least possible, though it may require a separation of gas between the nuclear explosive and the projectile, and probably a much weaker nuke than the likes of Fat Man (let alone Tsar Bomba).
    $endgroup$
    – Ingolifs
    3 hours ago










  • $begingroup$
    @Ingolifs the trouble with comparing Operation Plumbbob to this scenario is that it was conducted on Earth, not space. Earth has an atmosphere that can be heated as an expanding gas and plasma, space doesn't have a medium in which to propagate the blast wave.
    $endgroup$
    – Tim B II
    2 hours ago










  • $begingroup$
    I've edited my original question to include a mass of material between the projectile and the nuclear explosive, that vaporises into gas and plasma.
    $endgroup$
    – Ingolifs
    2 hours ago


















  • $begingroup$
    While I too question its practicality, is this not roughly analogous to the nuclear pulse rocket concept?
    $endgroup$
    – Cort Ammon
    6 hours ago










  • $begingroup$
    @CortAmmon you're right; I'd forgotten about that until you mentioned it. I just did a quick check on it and most of the material I can find on this talks about specific impulses, but doesn't actually go into detail about how the blast wave in space actually works. It seems to be related to the surrounding mass being converted to plasma. I'll do some more research and edit accordingly within the next couple of hours.
    $endgroup$
    – Tim B II
    6 hours ago










  • $begingroup$
    I'm not convinced by this answer. Operation Plumbbob has shown that such a thing is at least possible, though it may require a separation of gas between the nuclear explosive and the projectile, and probably a much weaker nuke than the likes of Fat Man (let alone Tsar Bomba).
    $endgroup$
    – Ingolifs
    3 hours ago










  • $begingroup$
    @Ingolifs the trouble with comparing Operation Plumbbob to this scenario is that it was conducted on Earth, not space. Earth has an atmosphere that can be heated as an expanding gas and plasma, space doesn't have a medium in which to propagate the blast wave.
    $endgroup$
    – Tim B II
    2 hours ago










  • $begingroup$
    I've edited my original question to include a mass of material between the projectile and the nuclear explosive, that vaporises into gas and plasma.
    $endgroup$
    – Ingolifs
    2 hours ago
















$begingroup$
While I too question its practicality, is this not roughly analogous to the nuclear pulse rocket concept?
$endgroup$
– Cort Ammon
6 hours ago




$begingroup$
While I too question its practicality, is this not roughly analogous to the nuclear pulse rocket concept?
$endgroup$
– Cort Ammon
6 hours ago












$begingroup$
@CortAmmon you're right; I'd forgotten about that until you mentioned it. I just did a quick check on it and most of the material I can find on this talks about specific impulses, but doesn't actually go into detail about how the blast wave in space actually works. It seems to be related to the surrounding mass being converted to plasma. I'll do some more research and edit accordingly within the next couple of hours.
$endgroup$
– Tim B II
6 hours ago




$begingroup$
@CortAmmon you're right; I'd forgotten about that until you mentioned it. I just did a quick check on it and most of the material I can find on this talks about specific impulses, but doesn't actually go into detail about how the blast wave in space actually works. It seems to be related to the surrounding mass being converted to plasma. I'll do some more research and edit accordingly within the next couple of hours.
$endgroup$
– Tim B II
6 hours ago












$begingroup$
I'm not convinced by this answer. Operation Plumbbob has shown that such a thing is at least possible, though it may require a separation of gas between the nuclear explosive and the projectile, and probably a much weaker nuke than the likes of Fat Man (let alone Tsar Bomba).
$endgroup$
– Ingolifs
3 hours ago




$begingroup$
I'm not convinced by this answer. Operation Plumbbob has shown that such a thing is at least possible, though it may require a separation of gas between the nuclear explosive and the projectile, and probably a much weaker nuke than the likes of Fat Man (let alone Tsar Bomba).
$endgroup$
– Ingolifs
3 hours ago












$begingroup$
@Ingolifs the trouble with comparing Operation Plumbbob to this scenario is that it was conducted on Earth, not space. Earth has an atmosphere that can be heated as an expanding gas and plasma, space doesn't have a medium in which to propagate the blast wave.
$endgroup$
– Tim B II
2 hours ago




$begingroup$
@Ingolifs the trouble with comparing Operation Plumbbob to this scenario is that it was conducted on Earth, not space. Earth has an atmosphere that can be heated as an expanding gas and plasma, space doesn't have a medium in which to propagate the blast wave.
$endgroup$
– Tim B II
2 hours ago












$begingroup$
I've edited my original question to include a mass of material between the projectile and the nuclear explosive, that vaporises into gas and plasma.
$endgroup$
– Ingolifs
2 hours ago




$begingroup$
I've edited my original question to include a mass of material between the projectile and the nuclear explosive, that vaporises into gas and plasma.
$endgroup$
– Ingolifs
2 hours ago


















draft saved

draft discarded




















































Thanks for contributing an answer to Worldbuilding Stack Exchange!


  • Please be sure to answer the question. Provide details and share your research!

But avoid



  • Asking for help, clarification, or responding to other answers.

  • Making statements based on opinion; back them up with references or personal experience.


Use MathJax to format equations. MathJax reference.


To learn more, see our tips on writing great answers.




draft saved


draft discarded














StackExchange.ready(
function () {
StackExchange.openid.initPostLogin('.new-post-login', 'https%3a%2f%2fworldbuilding.stackexchange.com%2fquestions%2f136933%2fsingle-use-nuke-propelled-mass-driver-spaceship%23new-answer', 'question_page');
}
);

Post as a guest















Required, but never shown





















































Required, but never shown














Required, but never shown












Required, but never shown







Required, but never shown

































Required, but never shown














Required, but never shown












Required, but never shown







Required, but never shown







Popular posts from this blog

Polycentropodidae

Magento 2 Error message: Invalid state change requested

Paulmy